LSAT and Law School Admissions Forum

Get expert LSAT preparation and law school admissions advice from PowerScore Test Preparation.

 Echx73
  • Posts: 36
  • Joined: Nov 11, 2015
|
#24547
Hey Team,

Can you write out the conditional statement for me, please? Lastly, I feel like B is worded too softly to be the correct answer of a conditional statement. When know from the stimulus lowering interest rates produces new capital. The correct answer uses the word "might" cause precondition for industrial growth. Thank you!

Eric
 David Boyle
PowerScore Staff
  • PowerScore Staff
  • Posts: 836
  • Joined: Jun 07, 2013
|
#24938
Echx73 wrote:Hey Team,

Can you write out the conditional statement for me, please? Lastly, I feel like B is worded too softly to be the correct answer of a conditional statement. When know from the stimulus lowering interest rates produces new capital. The correct answer uses the word "might" cause precondition for industrial growth. Thank you!

Eric

Hello Eric,

"Might" does seem a little of an understatement, but on a test where "some" can also mean "all", and "could be true" is true if something "must be true" (e.g., since I am writing this answer right now, it not only "must be true" that I'm doing it, it also "could be true"), "might" is good enough.

Diagrams:

Fully utilized :arrow: (growth :arrow: cap. invest)
Reduction :arrow: cap. invest.

Hope this helps,
David
 AlyonaHess
  • Posts: 1
  • Joined: Feb 03, 2018
|
#43503
Hi team,
Would you kindly explain the way you diagram the first condition? I’m confused with “no industrial growth without new capital investment”. Why did you negate the negation?

Thank you!
 Emily Haney-Caron
PowerScore Staff
  • PowerScore Staff
  • Posts: 577
  • Joined: Jan 12, 2012
|
#43539
Hi AlyonaHess,

Great question! You could also take the contrapositive of the diagram David made; capital investment :arrow: growth. The way David diagrammed it, it is a little easier to parse, but either way is correct. Just remember that for the contrapositive you're switching sides and negating both things. :)
 chian9010
  • Posts: 81
  • Joined: Jun 08, 2018
|
#46439
Dear Powerscore,

I have a question on answer choice A). Is this statement a correct description but this is not the best answer choice or not be able to concluded from the stimulus? I tried to find a flaw on this statement but I am not sure if there is any.

Thank you

CY
User avatar
 Jonathan Evans
PowerScore Staff
  • PowerScore Staff
  • Posts: 726
  • Joined: Jun 09, 2016
|
#46475
Hi, Chian,

Excellent question. In fact, answer choice (A) directly contradicts the stimulus. This is a Must Be True question, so we're looking for an answer choice that passes the Fact Test™. We need to find a statement that we can prove based on information in the stimulus. This stimulus contains two conditional statements:
  1. Industrial growth requires capital investment. IG :arrow: CI
  2. Reducing interest rates results in capital investment. RIR :arrow: CI
Thus, we know that any time we reduce interest rates, there must be capital investment. Answer choice (A) contradicts this statement. First, we don't know anything about "exceptional cases." Second, if we take the information in the stimulus to be true, even in exceptional cases there would still be capital investment every time interest rates are reduced.

Answer choice (B) gives us a scenario in which interest rates have been reduced. What do we know when interest rates are reduced? We know there will be capital investment. If he have capital investment, we then know that a necessary precondition for industrial growth has occurred.

If we use the conditional statements correctly, we actually can infer with more certainty that a precondition for industrial growth has been met with reduced interest rates. The reduction in interest rates "will produce" capital investment, not just "might cause."

In effect, we can say, "Hey guys, not only do we know the reduced interest rates might cause capital investment. They will cause capital investment." So we're super sure that (B) is a valid statement.

I hope this helps!
 S2KMo
  • Posts: 12
  • Joined: Jun 10, 2018
|
#46705
Jonathan Evans wrote:Hi, Chian,

Excellent question. In fact, answer choice (A) directly contradicts the stimulus. This is a Must Be True question, so we're looking for an answer choice that passes the Fact Test™. We need to find a statement that we can prove based on information in the stimulus. This stimulus contains two conditional statements:
  1. Industrial growth requires capital investment. IG :arrow: CI
  2. Reducing interest rates results in capital investment. RIR :arrow: CI
Thus, we know that any time we reduce interest rates, there must be capital investment. Answer choice (A) contradicts this statement. First, we don't know anything about "exceptional cases." Second, if we take the information in the stimulus to be true, even in exceptional cases there would still be capital investment every time interest rates are reduced.

Answer choice (B) gives us a scenario in which interest rates have been reduced. What do we know when interest rates are reduced? We know there will be capital investment. If he have capital investment, we then know that a necessary precondition for industrial growth has occurred.

If we use the conditional statements correctly, we actually can infer with more certainty that a precondition for industrial growth has been met with reduced interest rates. The reduction in interest rates "will produce" capital investment, not just "might cause."

In effect, we can say, "Hey guys, not only do we know the reduced interest rates might cause capital investment. They will cause capital investment." So we're super sure that (B) is a valid statement.

I hope this helps!

Hello, my only confusion with this explanation is that a necessary condition can occur without a sufficient condition occurring, so how is it that if we have reduced interest rates, we know that preconditions have been met for industrial growth? Because RIR :arrow: NCI and IG :arrow: NCI, but NCI can occur without either of those things (RIR & IG) occurring. Is there something I'm missing?
 James Finch
PowerScore Staff
  • PowerScore Staff
  • Posts: 943
  • Joined: Sep 06, 2017
|
#46720
Hi Mo,

"Precondition" here is used as a synonym for "necessary condition." New capital investments (NCI) is a necessary condition for both industrial growth (IG) and reducing interest rates (RIR), but because of the way the stimulus locates them chronologically, reducing interest rates would occur before new capital investment, which would then make it possible, but not definite, that industrial growth could occur. Answer choice (B) is describing this conditional relationship correctly, but in a way in which test takers may become confused by the chronology (assuming a Mistaken Reversal of the first conditional relationship). With conditional logic, the chronology is not important, just the necessary/sufficient relationships.

Hope this helps!
 S2KMo
  • Posts: 12
  • Joined: Jun 10, 2018
|
#46735
Ahh, makes sense!

Thanks James!
 onlywinter
  • Posts: 17
  • Joined: Apr 18, 2018
|
#49405
I was in between A and B.

I'm trying to understand why A is wrong.

I had diagrammed A as -RIR -> NCI. Because RIR is a sufficient but not necessary condition in RIR -> NCI according to the stimulus, negating RIR does not mean NCI would be negated. To assume otherwise, would turn the statement into a mistaken negation. I, therefore, thought A might happen. Where am I going wrong?

Get the most out of your LSAT Prep Plus subscription.

Analyze and track your performance with our Testing and Analytics Package.